PT1.S3.Q15 - Computer operating system software

Hinde SophiaHinde Sophia Alum Member
edited June 2022 in Logical Reasoning 165 karma

Hi fellow students! I'm a bit confused about the question listed above. I don't understand how, based on the language in the stimulus, we were supposed to infer anything about cost. Can someone help explain why B is the correct answer as opposed to C or E?

Admin Note: Edited title. Please use the format: "PT#.S#.Q# - brief description of the question"

Comments

  • CSieck3507CSieck3507 Member
    1376 karma

    So B is a subtle answer choice in my opinion. The overall conclusion is that we as a company should implement the variations. Why? Because it doesnt lose computer compatibility and also unauthorized access could be virtually eliminated. Okay, so you did give some good reasons as to why we should implement. The Q stem is asking us to strengthen so there must be some weakness. Okay, I am thinking of cost/benefit here. You have only given me two reasons as to why we should implement the varations. But what if they downside is greater compared to the 2 things that you gave? What if the variations do something to the system that the virus doesnt do? Another key thing to take away is the word "much". It states the vandal can destroy MUCH of the data. Okay but how much? We dont know! Our intuitions tell us that much means a lot, but we cant pinpoint an exact number. What if the variations make us lose data? I know common sense tells me that I can lose data with variations in the system while the variations are still compatible with the system. So lets get to strengthening. How can we show that the variations are still a stronger candidate? B gives another reason. Specifically dealing with the virus. If correcting the damage from the virus is more expensive than preventing the damage, then it lends credence to actually implement the variations! It goes back to this-What if the virus really isnt that bad? What if it is super fixable. We are never given this info but the stim makes us assume the virus is absolutely horrible and there is nothing we can do. But we cant assume this. Therefore, B gives us a reason to show that fixing the virus is actually a downside because of the COST to fix, and that is why we should use the variation instead.

    Sorry for the long winded response but I hope that helps!

Sign In or Register to comment.